Q20

 
ganbayou
Thanks Received: 0
Atticus Finch
Atticus Finch
 
Posts: 213
Joined: June 13th, 2015
 
 
 

Q20

by ganbayou Fri Jun 24, 2016 2:12 pm

Hi, I'm still not sure why A is wrong on Game#4 Q20...I thought it does not work neither because when W2, 2MM must be at 1/3...otherwise, it would be M2M4 and no place for WY or in another case, W with Y is at 2, MM would be at 1/3, so in either case M cannot be at 2...
Is it because it does not say how many grants can be awarded 1 quarter?
User avatar
 
ManhattanPrepLSAT1
Thanks Received: 1909
Atticus Finch
Atticus Finch
 
Posts: 2851
Joined: October 07th, 2009
 
 
 

Re: Q20

by ManhattanPrepLSAT1 Thu Jul 07, 2016 6:47 pm

So here's the diagram helping us see why (E) cannot be true:

Image

As for answer choice (A), M can go in Q2:

Image